Bounded growth of functions vs bounded growth of functions on countable sets

The name of the pictureThe name of the pictureThe name of the pictureClash Royale CLAN TAG#URR8PPP











up vote
2
down vote

favorite












I am wondering if the boundedness of growth can be characterized by sequences. I am not sure if I use the term "growth" correctly, or use the correct tags for this question. Here is what I mean.



Let $X$ be an uncountable set and let $F$ be a collection of real-valued functions on $X$ with the following property:



For every countable set $Asubset X$ there is a function $alpha_A:Ato(0,+infty) $ such that for every $fin F$ the set $leftfracf(x)alpha_A(x), xin Aright$ is bounded.




Does there exist a function $alpha:Xto(0,+infty)$ such that for every $fin F$ the set $leftfracf(x)alpha(x), xin Xright$ is bounded?




If the answer is "no", can we remedy the situation by assuming that $X$ is a topological space satisfying some mild conditions and $Fsubset C(X)$?










share|cite|improve this question





















  • If I do not misunderstand, then I think the answer is yes for easy reasons: applying your given condition in the special case where each A is a singleton, it follows that for each $xin X$ we have $h(x):=sup f(x) colon fin F<infty$. Then $h:Xto (0,infty)$ is your required upper bound for the collection $F$.
    – Yemon Choi
    2 hours ago






  • 3




    @YemonChoi: I think you do misunderstand --- the sup over $F$ is not supposed to be finite, just the sup over $A$ for each $f$ separately.
    – Nik Weaver
    2 hours ago










  • @YemonChoi it is exactly as Nik Weaver says. In fact, $F$ is WLOG a vector space, and the condition should be viewed as existence of a weighted supremum seminorm.
    – erz
    1 hour ago






  • 1




    @NikWeaver Thank you! Apologies to the OP.
    – Yemon Choi
    23 mins ago














up vote
2
down vote

favorite












I am wondering if the boundedness of growth can be characterized by sequences. I am not sure if I use the term "growth" correctly, or use the correct tags for this question. Here is what I mean.



Let $X$ be an uncountable set and let $F$ be a collection of real-valued functions on $X$ with the following property:



For every countable set $Asubset X$ there is a function $alpha_A:Ato(0,+infty) $ such that for every $fin F$ the set $leftfracf(x)alpha_A(x), xin Aright$ is bounded.




Does there exist a function $alpha:Xto(0,+infty)$ such that for every $fin F$ the set $leftfracf(x)alpha(x), xin Xright$ is bounded?




If the answer is "no", can we remedy the situation by assuming that $X$ is a topological space satisfying some mild conditions and $Fsubset C(X)$?










share|cite|improve this question





















  • If I do not misunderstand, then I think the answer is yes for easy reasons: applying your given condition in the special case where each A is a singleton, it follows that for each $xin X$ we have $h(x):=sup f(x) colon fin F<infty$. Then $h:Xto (0,infty)$ is your required upper bound for the collection $F$.
    – Yemon Choi
    2 hours ago






  • 3




    @YemonChoi: I think you do misunderstand --- the sup over $F$ is not supposed to be finite, just the sup over $A$ for each $f$ separately.
    – Nik Weaver
    2 hours ago










  • @YemonChoi it is exactly as Nik Weaver says. In fact, $F$ is WLOG a vector space, and the condition should be viewed as existence of a weighted supremum seminorm.
    – erz
    1 hour ago






  • 1




    @NikWeaver Thank you! Apologies to the OP.
    – Yemon Choi
    23 mins ago












up vote
2
down vote

favorite









up vote
2
down vote

favorite











I am wondering if the boundedness of growth can be characterized by sequences. I am not sure if I use the term "growth" correctly, or use the correct tags for this question. Here is what I mean.



Let $X$ be an uncountable set and let $F$ be a collection of real-valued functions on $X$ with the following property:



For every countable set $Asubset X$ there is a function $alpha_A:Ato(0,+infty) $ such that for every $fin F$ the set $leftfracf(x)alpha_A(x), xin Aright$ is bounded.




Does there exist a function $alpha:Xto(0,+infty)$ such that for every $fin F$ the set $leftfracf(x)alpha(x), xin Xright$ is bounded?




If the answer is "no", can we remedy the situation by assuming that $X$ is a topological space satisfying some mild conditions and $Fsubset C(X)$?










share|cite|improve this question













I am wondering if the boundedness of growth can be characterized by sequences. I am not sure if I use the term "growth" correctly, or use the correct tags for this question. Here is what I mean.



Let $X$ be an uncountable set and let $F$ be a collection of real-valued functions on $X$ with the following property:



For every countable set $Asubset X$ there is a function $alpha_A:Ato(0,+infty) $ such that for every $fin F$ the set $leftfracf(x)alpha_A(x), xin Aright$ is bounded.




Does there exist a function $alpha:Xto(0,+infty)$ such that for every $fin F$ the set $leftfracf(x)alpha(x), xin Xright$ is bounded?




If the answer is "no", can we remedy the situation by assuming that $X$ is a topological space satisfying some mild conditions and $Fsubset C(X)$?







fa.functional-analysis set-theory gn.general-topology real-analysis






share|cite|improve this question













share|cite|improve this question











share|cite|improve this question




share|cite|improve this question










asked 2 hours ago









erz

1,4881714




1,4881714











  • If I do not misunderstand, then I think the answer is yes for easy reasons: applying your given condition in the special case where each A is a singleton, it follows that for each $xin X$ we have $h(x):=sup f(x) colon fin F<infty$. Then $h:Xto (0,infty)$ is your required upper bound for the collection $F$.
    – Yemon Choi
    2 hours ago






  • 3




    @YemonChoi: I think you do misunderstand --- the sup over $F$ is not supposed to be finite, just the sup over $A$ for each $f$ separately.
    – Nik Weaver
    2 hours ago










  • @YemonChoi it is exactly as Nik Weaver says. In fact, $F$ is WLOG a vector space, and the condition should be viewed as existence of a weighted supremum seminorm.
    – erz
    1 hour ago






  • 1




    @NikWeaver Thank you! Apologies to the OP.
    – Yemon Choi
    23 mins ago
















  • If I do not misunderstand, then I think the answer is yes for easy reasons: applying your given condition in the special case where each A is a singleton, it follows that for each $xin X$ we have $h(x):=sup f(x) colon fin F<infty$. Then $h:Xto (0,infty)$ is your required upper bound for the collection $F$.
    – Yemon Choi
    2 hours ago






  • 3




    @YemonChoi: I think you do misunderstand --- the sup over $F$ is not supposed to be finite, just the sup over $A$ for each $f$ separately.
    – Nik Weaver
    2 hours ago










  • @YemonChoi it is exactly as Nik Weaver says. In fact, $F$ is WLOG a vector space, and the condition should be viewed as existence of a weighted supremum seminorm.
    – erz
    1 hour ago






  • 1




    @NikWeaver Thank you! Apologies to the OP.
    – Yemon Choi
    23 mins ago















If I do not misunderstand, then I think the answer is yes for easy reasons: applying your given condition in the special case where each A is a singleton, it follows that for each $xin X$ we have $h(x):=sup f(x) colon fin F<infty$. Then $h:Xto (0,infty)$ is your required upper bound for the collection $F$.
– Yemon Choi
2 hours ago




If I do not misunderstand, then I think the answer is yes for easy reasons: applying your given condition in the special case where each A is a singleton, it follows that for each $xin X$ we have $h(x):=sup f(x) colon fin F<infty$. Then $h:Xto (0,infty)$ is your required upper bound for the collection $F$.
– Yemon Choi
2 hours ago




3




3




@YemonChoi: I think you do misunderstand --- the sup over $F$ is not supposed to be finite, just the sup over $A$ for each $f$ separately.
– Nik Weaver
2 hours ago




@YemonChoi: I think you do misunderstand --- the sup over $F$ is not supposed to be finite, just the sup over $A$ for each $f$ separately.
– Nik Weaver
2 hours ago












@YemonChoi it is exactly as Nik Weaver says. In fact, $F$ is WLOG a vector space, and the condition should be viewed as existence of a weighted supremum seminorm.
– erz
1 hour ago




@YemonChoi it is exactly as Nik Weaver says. In fact, $F$ is WLOG a vector space, and the condition should be viewed as existence of a weighted supremum seminorm.
– erz
1 hour ago




1




1




@NikWeaver Thank you! Apologies to the OP.
– Yemon Choi
23 mins ago




@NikWeaver Thank you! Apologies to the OP.
– Yemon Choi
23 mins ago










1 Answer
1






active

oldest

votes

















up vote
3
down vote



accepted










This was a fun question! The answer is no.



Let $Omega$ be the set of all countable ordinals. For each limit ordinal $alpha in Omega$ let $f_alpha: Omega to [1,infty)$ be a function which increases to infinity on $[0,alpha)$ and is constantly zero on $[alpha,Omega)$.



For any limit ordinal $alpha in Omega$ the set of $f_beta$ with $beta leq alpha$ a limit ordinal is countable, and therefore there is a function $a_alpha: [0,alpha) to [1,infty)$ such that $f_beta preceq a_alpha$ for all $beta < alpha$ on $[0,alpha)$, where $preceq$ denotes "$leq$ except at finitely many points". Since any $f_beta$ with $beta > alpha$ is bounded on $[0,alpha)$ we get the stated condition.



However, there is no $a: Omega to (0,infty)$ that works. Given any such function $a$, there is an $n in mathbbN$ such that $a(alpha) leq n$ for uncountably many $alpha$. Let $(alpha_k)$ be an increasing sequence of such $alpha$'s and let $alpha$ be their supremum. Then the function $f_alpha$ goes to infinity on $[0,alpha)$ so its quotient with $a$ will still be unbounded.






share|cite|improve this answer




















  • Do I understand correctly that $a_alpha$ from your second can be constructed in the following way? Form a sequence $beta_n$ of the limit ordinals smaller than $alpha$ and then say that $a_alpha(beta)=max f_beta_k(beta)$, where $kle m$, where $beta_m$ is the the greatest limit ordinal smaller than $beta$.
    – erz
    1 hour ago







  • 1




    I think so, yes. It's easier to say more generally that any countable family of functions on a countable set can be majorized in this way: wlog the functions are $f_n: mathbbN to (0,infty)$, and then you let $a(n) = rm max(f_1(n), ldots, f_n(n))$.
    – Nik Weaver
    1 hour ago










Your Answer




StackExchange.ifUsing("editor", function ()
return StackExchange.using("mathjaxEditing", function ()
StackExchange.MarkdownEditor.creationCallbacks.add(function (editor, postfix)
StackExchange.mathjaxEditing.prepareWmdForMathJax(editor, postfix, [["$", "$"], ["\\(","\\)"]]);
);
);
, "mathjax-editing");

StackExchange.ready(function()
var channelOptions =
tags: "".split(" "),
id: "504"
;
initTagRenderer("".split(" "), "".split(" "), channelOptions);

StackExchange.using("externalEditor", function()
// Have to fire editor after snippets, if snippets enabled
if (StackExchange.settings.snippets.snippetsEnabled)
StackExchange.using("snippets", function()
createEditor();
);

else
createEditor();

);

function createEditor()
StackExchange.prepareEditor(
heartbeatType: 'answer',
convertImagesToLinks: true,
noModals: false,
showLowRepImageUploadWarning: true,
reputationToPostImages: 10,
bindNavPrevention: true,
postfix: "",
noCode: true, onDemand: true,
discardSelector: ".discard-answer"
,immediatelyShowMarkdownHelp:true
);



);













 

draft saved


draft discarded


















StackExchange.ready(
function ()
StackExchange.openid.initPostLogin('.new-post-login', 'https%3a%2f%2fmathoverflow.net%2fquestions%2f311524%2fbounded-growth-of-functions-vs-bounded-growth-of-functions-on-countable-sets%23new-answer', 'question_page');

);

Post as a guest






























1 Answer
1






active

oldest

votes








1 Answer
1






active

oldest

votes









active

oldest

votes






active

oldest

votes








up vote
3
down vote



accepted










This was a fun question! The answer is no.



Let $Omega$ be the set of all countable ordinals. For each limit ordinal $alpha in Omega$ let $f_alpha: Omega to [1,infty)$ be a function which increases to infinity on $[0,alpha)$ and is constantly zero on $[alpha,Omega)$.



For any limit ordinal $alpha in Omega$ the set of $f_beta$ with $beta leq alpha$ a limit ordinal is countable, and therefore there is a function $a_alpha: [0,alpha) to [1,infty)$ such that $f_beta preceq a_alpha$ for all $beta < alpha$ on $[0,alpha)$, where $preceq$ denotes "$leq$ except at finitely many points". Since any $f_beta$ with $beta > alpha$ is bounded on $[0,alpha)$ we get the stated condition.



However, there is no $a: Omega to (0,infty)$ that works. Given any such function $a$, there is an $n in mathbbN$ such that $a(alpha) leq n$ for uncountably many $alpha$. Let $(alpha_k)$ be an increasing sequence of such $alpha$'s and let $alpha$ be their supremum. Then the function $f_alpha$ goes to infinity on $[0,alpha)$ so its quotient with $a$ will still be unbounded.






share|cite|improve this answer




















  • Do I understand correctly that $a_alpha$ from your second can be constructed in the following way? Form a sequence $beta_n$ of the limit ordinals smaller than $alpha$ and then say that $a_alpha(beta)=max f_beta_k(beta)$, where $kle m$, where $beta_m$ is the the greatest limit ordinal smaller than $beta$.
    – erz
    1 hour ago







  • 1




    I think so, yes. It's easier to say more generally that any countable family of functions on a countable set can be majorized in this way: wlog the functions are $f_n: mathbbN to (0,infty)$, and then you let $a(n) = rm max(f_1(n), ldots, f_n(n))$.
    – Nik Weaver
    1 hour ago














up vote
3
down vote



accepted










This was a fun question! The answer is no.



Let $Omega$ be the set of all countable ordinals. For each limit ordinal $alpha in Omega$ let $f_alpha: Omega to [1,infty)$ be a function which increases to infinity on $[0,alpha)$ and is constantly zero on $[alpha,Omega)$.



For any limit ordinal $alpha in Omega$ the set of $f_beta$ with $beta leq alpha$ a limit ordinal is countable, and therefore there is a function $a_alpha: [0,alpha) to [1,infty)$ such that $f_beta preceq a_alpha$ for all $beta < alpha$ on $[0,alpha)$, where $preceq$ denotes "$leq$ except at finitely many points". Since any $f_beta$ with $beta > alpha$ is bounded on $[0,alpha)$ we get the stated condition.



However, there is no $a: Omega to (0,infty)$ that works. Given any such function $a$, there is an $n in mathbbN$ such that $a(alpha) leq n$ for uncountably many $alpha$. Let $(alpha_k)$ be an increasing sequence of such $alpha$'s and let $alpha$ be their supremum. Then the function $f_alpha$ goes to infinity on $[0,alpha)$ so its quotient with $a$ will still be unbounded.






share|cite|improve this answer




















  • Do I understand correctly that $a_alpha$ from your second can be constructed in the following way? Form a sequence $beta_n$ of the limit ordinals smaller than $alpha$ and then say that $a_alpha(beta)=max f_beta_k(beta)$, where $kle m$, where $beta_m$ is the the greatest limit ordinal smaller than $beta$.
    – erz
    1 hour ago







  • 1




    I think so, yes. It's easier to say more generally that any countable family of functions on a countable set can be majorized in this way: wlog the functions are $f_n: mathbbN to (0,infty)$, and then you let $a(n) = rm max(f_1(n), ldots, f_n(n))$.
    – Nik Weaver
    1 hour ago












up vote
3
down vote



accepted







up vote
3
down vote



accepted






This was a fun question! The answer is no.



Let $Omega$ be the set of all countable ordinals. For each limit ordinal $alpha in Omega$ let $f_alpha: Omega to [1,infty)$ be a function which increases to infinity on $[0,alpha)$ and is constantly zero on $[alpha,Omega)$.



For any limit ordinal $alpha in Omega$ the set of $f_beta$ with $beta leq alpha$ a limit ordinal is countable, and therefore there is a function $a_alpha: [0,alpha) to [1,infty)$ such that $f_beta preceq a_alpha$ for all $beta < alpha$ on $[0,alpha)$, where $preceq$ denotes "$leq$ except at finitely many points". Since any $f_beta$ with $beta > alpha$ is bounded on $[0,alpha)$ we get the stated condition.



However, there is no $a: Omega to (0,infty)$ that works. Given any such function $a$, there is an $n in mathbbN$ such that $a(alpha) leq n$ for uncountably many $alpha$. Let $(alpha_k)$ be an increasing sequence of such $alpha$'s and let $alpha$ be their supremum. Then the function $f_alpha$ goes to infinity on $[0,alpha)$ so its quotient with $a$ will still be unbounded.






share|cite|improve this answer












This was a fun question! The answer is no.



Let $Omega$ be the set of all countable ordinals. For each limit ordinal $alpha in Omega$ let $f_alpha: Omega to [1,infty)$ be a function which increases to infinity on $[0,alpha)$ and is constantly zero on $[alpha,Omega)$.



For any limit ordinal $alpha in Omega$ the set of $f_beta$ with $beta leq alpha$ a limit ordinal is countable, and therefore there is a function $a_alpha: [0,alpha) to [1,infty)$ such that $f_beta preceq a_alpha$ for all $beta < alpha$ on $[0,alpha)$, where $preceq$ denotes "$leq$ except at finitely many points". Since any $f_beta$ with $beta > alpha$ is bounded on $[0,alpha)$ we get the stated condition.



However, there is no $a: Omega to (0,infty)$ that works. Given any such function $a$, there is an $n in mathbbN$ such that $a(alpha) leq n$ for uncountably many $alpha$. Let $(alpha_k)$ be an increasing sequence of such $alpha$'s and let $alpha$ be their supremum. Then the function $f_alpha$ goes to infinity on $[0,alpha)$ so its quotient with $a$ will still be unbounded.







share|cite|improve this answer












share|cite|improve this answer



share|cite|improve this answer










answered 1 hour ago









Nik Weaver

17.9k143114




17.9k143114











  • Do I understand correctly that $a_alpha$ from your second can be constructed in the following way? Form a sequence $beta_n$ of the limit ordinals smaller than $alpha$ and then say that $a_alpha(beta)=max f_beta_k(beta)$, where $kle m$, where $beta_m$ is the the greatest limit ordinal smaller than $beta$.
    – erz
    1 hour ago







  • 1




    I think so, yes. It's easier to say more generally that any countable family of functions on a countable set can be majorized in this way: wlog the functions are $f_n: mathbbN to (0,infty)$, and then you let $a(n) = rm max(f_1(n), ldots, f_n(n))$.
    – Nik Weaver
    1 hour ago
















  • Do I understand correctly that $a_alpha$ from your second can be constructed in the following way? Form a sequence $beta_n$ of the limit ordinals smaller than $alpha$ and then say that $a_alpha(beta)=max f_beta_k(beta)$, where $kle m$, where $beta_m$ is the the greatest limit ordinal smaller than $beta$.
    – erz
    1 hour ago







  • 1




    I think so, yes. It's easier to say more generally that any countable family of functions on a countable set can be majorized in this way: wlog the functions are $f_n: mathbbN to (0,infty)$, and then you let $a(n) = rm max(f_1(n), ldots, f_n(n))$.
    – Nik Weaver
    1 hour ago















Do I understand correctly that $a_alpha$ from your second can be constructed in the following way? Form a sequence $beta_n$ of the limit ordinals smaller than $alpha$ and then say that $a_alpha(beta)=max f_beta_k(beta)$, where $kle m$, where $beta_m$ is the the greatest limit ordinal smaller than $beta$.
– erz
1 hour ago





Do I understand correctly that $a_alpha$ from your second can be constructed in the following way? Form a sequence $beta_n$ of the limit ordinals smaller than $alpha$ and then say that $a_alpha(beta)=max f_beta_k(beta)$, where $kle m$, where $beta_m$ is the the greatest limit ordinal smaller than $beta$.
– erz
1 hour ago





1




1




I think so, yes. It's easier to say more generally that any countable family of functions on a countable set can be majorized in this way: wlog the functions are $f_n: mathbbN to (0,infty)$, and then you let $a(n) = rm max(f_1(n), ldots, f_n(n))$.
– Nik Weaver
1 hour ago




I think so, yes. It's easier to say more generally that any countable family of functions on a countable set can be majorized in this way: wlog the functions are $f_n: mathbbN to (0,infty)$, and then you let $a(n) = rm max(f_1(n), ldots, f_n(n))$.
– Nik Weaver
1 hour ago

















 

draft saved


draft discarded















































 


draft saved


draft discarded














StackExchange.ready(
function ()
StackExchange.openid.initPostLogin('.new-post-login', 'https%3a%2f%2fmathoverflow.net%2fquestions%2f311524%2fbounded-growth-of-functions-vs-bounded-growth-of-functions-on-countable-sets%23new-answer', 'question_page');

);

Post as a guest













































































Comments

Popular posts from this blog

Long meetings (6-7 hours a day): Being “babysat” by supervisor

Is the Concept of Multiple Fantasy Races Scientifically Flawed? [closed]

Confectionery